Given enough time, everyone is capable of computing \(x^3-2x^2+7x+11\) if say \(x=1.1\); all operations reduce to addition/multiplication of rational numbers. But have you ever asked yourself how you actually compute say \(\sin(1.1)\) or \(e^{1.1}\) or maybe \(\arctan(1.1)\)? It turns out that we have no way to compute \(\sin(1.1)\) exactly (and similarly \(e^{1.1}\) and \(\arctan(1.1)\)), even with the world's fastest super computer with all the available memory. We can, however, approximate such quantities and even determine a bound on the error of our approximation. How, you ask? Well, one of the main goals of this part of the course is to show that many familiar functions can be represented as an infinite sum of polynomials. For example, we will show that for all \(x\) it is true that \begin{align*} e^x &= 1 + x+ \frac{x^2}{2!} + \frac{x^3}{3!} + \frac{x^4}{4!} + \cdots \\[2ex] \sin(x) &= x - \frac{x^3}{3!} + \frac{x^5}{5!} - \frac{x^7}{7!} + \frac{x^9}{9!} - \cdots \\[2ex] \arctan(x) &= x - \frac{x^3}{3} + \frac{x^5}{5} - \frac{x^7}{7} + \frac{x^9}{9} - \cdots \end{align*} One of the uses of representing a function with an infinite sum of polynomials is that we can truncate the infinite sum to obtain an approximation using a finite sum of polynomials. For example, \[ e^x \approx 1 + x+ \frac{x^2}{2!} + \frac{x^3}{3!} + \frac{x^4}{4!} + \frac{x^5}{5!}. \] How else do you think your calculator computes \(e^x\)? Consider the expansion for \(\arctan(x)\). Since \(\arctan(1) = \frac{\pi}{4}\), then by plugging in \(x=1\) in the representation of \(\arctan(x)\) we obtain that \[ \frac{\pi}{4} = 1 - \frac{1}{3} + \frac{1}{5} - \frac{1}{7} + \frac{1}{9} - \frac{1}{11} + \cdots \] and therefore multiplying both sides by \(4\) we obtain \[ \pi = 4 - \frac{4}{3} + \frac{4}{5} - \frac{4}{7} + \frac{4}{9} - \frac{4}{11} + \cdots \] Or consider the expansion for \(e^x\). Plugging in \(x=1\) into the representation of \(e^x\) we obtain \[ e = 1 + 1 + \frac{1}{2!} + \frac{1}{3!} + \frac{1}{4!} + \frac{1}{5!} + \cdots \] Before we can understand how to represent a function as a sum of infinite polynomials, we need to understand what it means for an infinite sum of real numbers to ``sum-up'' to a finite number. Infinite sums are called infinite series or just series, and to understand series we need to understand convergence of sequences.

Sequences

A sequence is a list of numbers in a given order: \[ a=(a_1,a_2,a_3,a_4,\ldots) \] Here are some sequences: \begin{align*} a &= (2,4,6,8,10,\ldots)\\[2ex] a &= \left(1,\frac{1}{2},\frac{1}{4},\frac{1}{8},\ldots,\right)\\[2ex] a &= (3, 3.1, 3.14, 3.141, 3.1415, 3.14159,\ldots)\\[2ex] a &= \left(-2,\frac{3}{2},-\frac{4}{3},\frac{5}{4},-\frac{6}{5},\ldots\right) \end{align*} The \(n\)th term of a sequence \(a\) is denoted by \(a_n\). For instance, in the last sequence above, \(a_1=-2\), \(a_2=\frac{3}{2}\), \(a_3=-\frac{4}{3}\), and so on. The integer \(n\) is called the index of the sequence and usually the index starts at \(n=1\) and sometimes at \(n=0\). A sequence may be defined using a formula for the general \(n\)th term \(a_n\). For example, the sequence of even numbers \(a=(2,4,6,8,10,\ldots)\) has general \(n\)th term \[ a_n = 2n, \qquad n=1,2,3,4,\ldots \] The sequence \(a=(1,1/2, 1/4, 1/8, \ldots)\) has general \(n\)th term \[ a_n = \frac{1}{2^{n-1}}, \qquad n=1,2,3,4,\ldots \] Notice that for this last sequence, we could write the general term \(a_n\) in the form \[ a_n = \frac{1}{2^n},\qquad n=0,1,2,3,\ldots \] which is aesthetically more pleasing to the eye.
Write out the first few terms of the given sequence \(a=(a_1,a_2,a_3,\ldots)\)
  1. \(a_n = \frac{(-1)^n n}{n^2-1}\)
  2. \(a_n = \frac{2n+1}{n!}\)
Some general notation you might see for denoting a sequence is \(\{a_n\}_{n=1}^\infty\), \(\{a_n\}_{n=0}^\infty\), \((a_n)_{n=1}^\infty\), etc. From now on we will use \((a_n)\).

Convergence of Sequences

Before we discuss convergence of sequences, we review some basics regarding absolute value and inequalities. First of all, the inequality \(|y| \lt 3\) means that \(-3 \lt y \lt 3\), or \(y\) lies in the interval \((-3,3)\). Given numbers \(x\) and \(L\), the number \(|x-L|\) is the distance between \(x\) and \(L\). Therefore, if \(|x-L| \lt \eps\) then the distance between \(x\) and \(L\) is less than \(\eps\): \[ -\eps \lt x- L \lt \eps \] and adding \(L\) to all terms \[ L - \eps \lt x \lt L + \eps. \] Hence, \(x\) lies in the interval \((L-\eps, L+\eps)\). Consider the sequence \((a_n)\) with \(n\)th term \(a_n = 1 + \frac{(-1)^n}{2^n}\). The first few terms are \[ a = (\tfrac{1}{2}, \tfrac{5}{4}, \tfrac{7}{8}, \tfrac{17}{16}, \ldots) \] Here is a picture:
figures/sequence-on-real-line.svg
We will show that for \(n\) sufficiently large, the numbers \(a_n\) get arbitrarily close to \(1\). In other words, for any given small quantity \(\varepsilon \gt 0\), the terms of the sequence from some point and onward will lie in the interval \[ (1-\eps,1+\eps). \] In other words, for \(n\) sufficiently large it holds that \[ |a_n - 1| \lt \varepsilon. \] For example, if say \(\eps=0.0000001\) then for which \(n\) is it true that \(|a_n - 1| \lt \eps\)? Consider first \begin{align*} |a_n - 1| = \left| 1+\frac{(-1)^n}{2^n} - 1 \right| = \frac{1}{2^n}. \end{align*} Now \(\frac{1}{2^n} \lt \eps\) when \[ n \gt \frac{\ln(1/\eps)}{\ln(2)} \approx 23.25. \] Therefore, if \(n\geq 24\) then \(|a_n-1| \lt \eps\) or \(1-\eps \lt a_n \lt 1+\eps\). In fact, \begin{align*} a_{24} &= 1.0000000596046448\\ a_{25} &= 0.9999999701976776\\ a_{26} &= 1.0000000149011612 \end{align*} If instead \(\eps = 1\times 10^{-16}\) then need \(n\geq 54\) for \(a_n\) to lie in the smaller interval \((1-\eps,1+\eps)\). However, no matter how small \(\eps\) is, there exists some natural number \(M\) such that if \(n\geq M\) then \(a_n\) lies in the interval \((1-\eps,1+\eps)\).
A sequence \((a_n)\) is said to converge if there exists a number \(L\) such that for any given \(\eps \gt 0\) there exists a natural number \(M\) such that if \(n\geq M\) then \(|a_n - L| \lt \eps\). In this case, we then write that \[ \lim_{n\rightarrow\infty} a_n = L \] and call \(L\) the limit of \(a_n\).
Show by definition that the sequence \(a_n = \frac{n-1}{2n+1}\) converges to \(L=\frac{1}{2}\). Find the smallest number \(M\) such that if \(n\geq M\) then \(|a_n - \tfrac{1}{2}| \lt 0.0001\).
Let \(\eps \gt 0\) be arbitrary. Now \[ |a_n - \tfrac{1}{2}| = \left|\frac{2n-2-2n-1}{2n+1} \right| = \left|\frac{-3}{2(2n+1)}\right| = \frac{3/2}{2n+1} \] Now \[ \frac{3/2}{2n+1} \lt \eps \] when \[ \frac{3}{4\eps}-\frac{1}{2} \lt n \] Hence, need \(M \gt \frac{3}{4\eps}-\frac{1}{2}=7499.5\). The minimal \(M\) is thus \(M=7500\).
A sequence \((a_n)\) is said to be bounded if all the terms of the sequence are contained in some interval, in other words, there exists \(c,d\in\real\) such that \(c\leq a_n \leq d\) for all \(n\). If \((a_n)\) is bounded then does \((a_n)\) necessarily converge? Not necessarily, for example consider the sequences \[ (b_n) = (1, -1, 1, -1, 1, -1, \ldots),   b_n = \frac{(-1)^n (n+1)}{n} \] If \((a_n)\) is convergent then is \((a_n)\) necessarily bounded? Yes! For any \(\eps \gt 0\), there exists \(M\) such that \(L-\eps \lt a_n \lt L+\eps\) for all \(n\geq M\). The other terms \(a_1, a_2, \ldots, a_{M-1}\) may not be in the interval \((L-\eps, L+\eps)\) but there is only finitely many of them. As an example, consider \[ a_n = (-1)^n \sqrt{n}. \] Then \(|a_n| = \sqrt{n}\) and this gets arbitrarily large and thus \((a_n)\) is not bounded; therefore \((a_n)\) does not converge. To compute a limit \[ \limi a_n \] we can compute it as if we were computing a functional limit of the form \[ \lim_{x\rightarrow\infty} f(x) \] and we can even use l'H\^{o}pital's rule. Given sequences \((a_n)\) and \((b_n)\), we can obtain a new sequence \((c_n)\) given by \(c_n = a_n + b_n\). In other words, \[ (c_n) = (a_n+b_n) = (a_1+b_1, a_2 + b_2, a_3+b_3, \ldots) \] We can also define the difference, product, division, and scalar multiplication of sequences: \begin{align*} c_n &= a_n - b_n \\ c_n &= a_n b_n \\ c_n &= \frac{a_n}{b_n},  \text{provided \(b_n\neq 0\)}\\ c_n &= k a_n,  \text{\(k\) a constant} \end{align*}
Suppose that \(\ds\lim_{n\rightarrow\infty}a_n\) and \(\ds\lim_{n\rightarrow\infty} b_n \) exist. Then:
  1. \(\ds\lim_{n\rightarrow\infty} (a_n+b_n) = \limi a_n + \limi b_n\)
  2. \(\ds\limi (a_n - b_n) = \limi a_n - \limi b_n\)
  3. \(\ds\limi (k a_n) = k \ds\limi a_n\)
  4. \(\ds\limi (a_n b_n) = \left(\ds\limi a_n\right) \left(\ds\limi b_n\right)\)
  5. If \(b_n\neq 0\) and \(\ds\limi b_n \neq 0\) then \[ \limi \left(\frac{a_n}{b_n}\right) = \frac{\ds\limi a_n}{\ds\limi b_n} \]
Compute the following limits.
  1. \(\ds\limi \frac{37+3n-n^3}{4+n^2+3n^3} \) (Do it by factoring \(n^3\))
  2. \(\ds\limi \sqrt{\frac{3n^2+7}{4n^2+n-1}} \) (Bring limit inside)
  3. \(\ds\limi \cos\left(\frac{\pi n+1}{2n}\right) \) (Bring limit inside)
  4. \(\ds\limi \frac{\ln(n)}{n} \) (H.R.)
We must be careful how we apply the Limit Laws. For example, consider \(a_n = \frac{\sin(n)}{n^2}\). Then we might be tempted to write \begin{align*} \lim_{n\rightarrow\infty} \frac{\sin(n)}{n^2} &= \limi \frac{1}{n^2} \sin(n)\\ &= \left(\limi \frac{1}{n^2}\right)\left( \limi \sin(n)\right) \\ &= 0 \cdot \limi \sin(n) = 0 \end{align*} However, \(\limi \sin(n)\) does not exist and thus we cannot apply the Limit Laws. In this case, we must use the following.
Suppose that \(a_n \leq b_n \leq c_n\) for all \(n\). If \(\ds\limi a_n = L\) and \(\ds\limi c_n = L\) then \(\ds\limi b_n = L\).
Compute the following limits.
  1. \(\ds\limi \frac{\sin(n)}{n^2} \) (Squeeze)
  2. \(\ds\limi \frac{(-1)^n(\cos^2(n)+1)}{e^n} \) (Squeeze)
Find \(\ds\limi \left(\frac{n+1}{n-1}\right)^n.\)
Get an indeterminate form of the type \(1^\infty\). Consider then \begin{align*} \limi \ln \left(\frac{n+1}{n-1}\right)^n &= \limi n \ln \left(\frac{n+1}{n-1}\right)\\ &= \limi \frac{\ln \left(\frac{n+1}{n-1}\right)}{1/n} \\[2ex] &= \limi \frac{\frac{1}{n+1}-\frac{1}{n-1}}{-1/n^2}\\[2ex] &= \limi \frac{2n^2}{n^2-1} = 2 \end{align*} Hence, \(\ds\limi \left(\frac{n+1}{n-1}\right)^n = e^2\).
Find \(\ds\limi \left(\frac{3}{4}\right)^n.\)
Write as \begin{align*} \limi \left(\frac{3}{4}\right)^n &= \limi e^{\ln (3/4)^n}\\ &= \limi e^{ \ln(3/4) n} \end{align*} Because \(\ln(3/4) \lt 0\) then \(\ds\limi e^{\ln(3/4)n} = 0\). Hence, \(\ds\limi (3/4)^n = 0\). In fact, for any number \( -1 \lt p \lt 1\) it holds that \[ \limi p ^n = 0. \] If \(|p| \gt 1\) then \(\ds\limi p^n\) does not exist.
A sequence \((a_n)\) is increasing if \[ a_1 \leq a_2 \leq a_3 \leq \cdots \] and is called decreasing if \[ a_1 \geq a_2 \geq a_3 \geq \cdots \] A sequence that is either increasing or decreasing is called monotone. If \((a_n)\) is convergent is it necessarily increasing or decreasing? Not necessarily, for example consider the sequence \(a_n = \frac{(-1)^n}{n}\) or \[ (a_n) = \left(-1, \frac{1}{2}, -\frac{1}{3}, \frac{1}{4}, \cdots \right) \] It is fairly clear that \(\ds\lim a_n = 0\) but \((a_n)\) is not monotone. We now know that a sequence that is bounded is not necessarily convergent. But what about a sequence that is monotone and bounded?
If \((a_n)\) is bounded and monotone then \((a_n)\) is convergent.
Consider the sequence \((s_n)\) whose general \(n\)th term is \[ s_n = 1 + \frac{1}{1!} + \frac{1}{2!} + \cdots + \frac{1}{n!} \] Notice that \[ s_{n+1} = 1 + \frac{1}{1!} + \frac{1}{2!} + \cdots + \frac{1}{n!} + \frac{1}{(n+1)!} = s_n + \frac{1}{(n+1)!} \] It is now clear that \(s_n \lt s_{n+1}\) for any \(n\). Therefore, \((s_n)\) is increasing. It can be shown that \((s_n)\) is bounded. Therefore, by the MCT, \(\ds\limi s_n\) exists.

Infinite Series

A sequence \(a=(a_1,a_2,a_3,\ldots)\) gives rise to a new sequence \(s=(s_1,s_2,s_3,\ldots)\) called the sequence of partial sums of \(a\) defined as follows: \begin{align*} s_1 &= a_1 \\ s_2 &= a_1 + a_2 \\ s_3 &= a_1+a_2 + a_3\\ \vdots\; &\qquad\qquad \vdots\\ s_k &= a_1 + a_2 + \cdots + a_k \end{align*} Using summation convention, the \(k\)th term of \(s=(s_1,s_2,s_3,\ldots)\) is therefore \[ s_k = \sum_{n=1}^k a_n. \] Notice that \[ s_{k+1} = \underbrace{a_1 + a_2 + \cdots + a_k}_{s_k} + a_{k+1} = s_k + a_{k+1}. \]
Let \(a_n = \frac{1}{2^{n-1}}\) for \(n=1,2,3,\ldots\). Find the first ten terms of the sequence of partial sums associated to \((a_n)\).
We compute \begin{align*} s_1 &= a_1 = 1\\ s_2 &= a_1 + a_2 = 1 + \tfrac{1}{2} = \tfrac{3}{2}=1.5\\ s_3 &= a_1 + a_2 + a_3 = \tfrac{3}{2} + \tfrac{1}{4} = \tfrac{7}{4}=1.75\\ s_4 &= \tfrac{7}{4} + \tfrac{1}{8} = \tfrac{15}{8}=1.875\\ s_5 &= s_4 + \tfrac{1}{16} = 1.9375\\ s_6 &= s_5 + \tfrac{1}{32} = 1.96875\\ s_7 &= s_6 + \tfrac{1}{64} = 1.984375\\ s_8 &= s_7 + \tfrac{1}{128} = 1.9921875\\ s_9 &= s_8 + \tfrac{1}{256} = 1.99609375\\ s_{10} &= s_9 + \tfrac{1}{512} = 1.998046875 \end{align*} In fact, with a bit more patience one can compute that \begin{align*} s_{20} &= 1.9999980926513672\\ s_{50} &= 1.9999999999999982 \end{align*} It seems like \(\ds\lim_{k\rightarrow\infty} s_k = 2\).
We will be interested in computing limits of sequences of partial sums. We define an infinite series as the limit of the sequence of partial sums of a given sequence, and we use the following notation for the limit: \[ \lim_{k\rightarrow\infty} s_k = \lim_{k\rightarrow\infty} \left(\sum_{n=1}^k a_n \right) = \sum_{n=1}^\infty a_n \] Hence, our previous example suggests that \[ \sum_{n=1}^\infty \frac{1}{2^{n-1}} = 2 \] or using expanded notation \[ 1 + \frac{1}{2} + \frac{1}{2^2} + \frac{1}{2^3} + \cdots = 2 \] In general, if \(\ds\limik s_k = \sum_{n=1}^\infty a_n\) exists then we say that the infinite series \(\sum_{n=1}^\infty a_n\) converges, otherwise we say that it diverges.
Determine whether the infinite series generated by the sequence \(a=(1,2,3,4,\ldots)\) converges.
Here \(a_n = n\) and thus the sequence of partial sums is \[ s_k = a_1 + a_2 + \cdots + a_k = 1 + 2 + 3 + \cdots + k \] Here are the first few terms of \(s=(s_1,s_2,s_3,s_4,\ldots)\): \[ s_1 = 1,\; s_2 = 3,\; s_3 = 6,\; s_4 = 10,\; s_5 = 15 \] It seems as though \(s\) is divergent because it is unbounded. To see this: \begin{align*} s_k &= 1 + 2 + 3 + \cdots + k \\ s_k &= k + (k-1) + (k-2) + \cdots + 1 \end{align*} and thus \[ 2 s_k = k(k+1)  \Longrightarrow   s_k = \frac{k(k+1)}{2} \] For instance, \(s_5 = 5\cdot 6 / 2 = 15\) and agrees with our previous computation. It is clear that \[ \limik s_k = \limik \frac{k(k+1)}{2} \] does not exist. Hence, the infinite series \(\sum_{n=1}^\infty a_n = \sum_{n=1}^\infty n\) diverges.
Let \(r\) be a constant and let \(a_n = r^{n}\) for \(n=0,1,2,3,\ldots\). Investigate the convergence of the infinite series generated by \((a_n)\).
The infinite series is \[ \sum_{n=0}^\infty r^{n} = 1 + r + r^2 + r^3 + \cdots \] The sequence of partial sums \((s_n)\) has general \(k\)th term \[ s_k = \sum_{n=0}^k r^{n} = 1 + r + r^2 + \cdots + r^{k}. \] We can get a closed-form formula for \(s_k\) by using \[ (1-r)(1+r+r^2+\cdots+r^{k}) = 1-r^{k+1} \] and thus we obtain that \[ s_k = 1+r+r^2+\cdots+r^{k} = \frac{1-r^{k+1}}{1-r} \] We need to consider the limit \(\ds\lim_{k\rightarrow\infty} s_k\), which amounts to determining \(\ds\lim_{k\rightarrow\infty} r^k\): \[ \lim_{k\rightarrow\infty} s_k = \lim_{k\rightarrow\infty} \frac{1-r^{k+1}}{1-r} = \begin{cases} \frac{1}{1-r}, & |r| \lt 1\\[2ex] \text{DNE}, & |r|\geq 1 \end{cases} \] Therefore, the series \(\sumz r^{n}\) converges when \(|r| \lt 1\) and it converges to \[ \sumz r^{n} = \frac{1}{1-r}. \] This series is the most important series in this course; it is called the GEOMETRIC SERIES. Notice that we have \[ \sumz r^n = \sumo r^{n-1} \]
Determine whether the given series converges, and if yes, determine the value of the convergent series if possible.
  1. \(\ds\sumz \frac{3^n}{4^n}=\frac{1}{1-3/4}\)
  2. \(\ds\sumz \frac{(-1)^n}{4^n}=\frac{1}{1+(1/4)}\)
  3. \(\ds\sumz \frac{1}{7^n}=\frac{1}{1-1/7}\)
  4. \(\ds\sumz \frac{(-3)^n}{2^n}\) (DNE)
Use a geometric series to show that \[ 0.999999999 \cdots = 1 \]
Below is a test for divergence of a series.
Consider the series \(\sum a_n\). If \(\ds\limi a_n \neq 0\) then the series \(\sum a_n\) diverges. In other words, if \(\sum a_n\) converges then it must hold that \(\ds\limi a_n = 0\).
Using the divergence test, we can easily deduce that following series diverge:
  1. \(\ds\sumo 5n\)
  2. \(\ds\sumo n^2\)
  3. \(\ds\sumo \frac{2n+1}{3n+7}\)
  4. \(\ds\sumo \cos(\tfrac{1}{n})\)
In examples (i)-(ii), the limits \(\limi a_n\) do not exist. In case (ii), \(\limi a_n =2/3\neq 0\), and in case (iv) \(\limi a_n = 1\neq 0\).
Consider the series \[ \sumo \frac{1}{n} \] In this case, \(a_n = \frac{1}{n}\) and \(\ds\limi a_n = 0\). However, we cannot conclude that the given series converges. In fact, it can be shown that the sequence of partials sums \((s_n)\) associated to the sequence \((a_n)\) is unbounded. Therefore, \((s_n)\) diverges and consequently the given series diverges.
We now state some basic arithmetic properties of infinite series.
Suppose that \(\sum a_n\) and \(\sum b_n\) are convergent. Then
  1. \(\sum (a_n + b_n)\) converges and \(\sum (a_n+b_n) = \sum a_n + \sum b_n\)
  2. \(\sum (a_n - b_n)\) converges and \(\sum (a_n-b_n) = \sum a_n - \sum b_n\)
  3. For any constant \(c\), \(\sum ca_n\) converges and \(\sum c a_n = c \sum a_n\)
Important: We note that if \(\sum a_n\) converges and \(\sum b_n\) diverges then \(\sum (a_n \pm b_n)\) diverges.
Determine if the given series converge and if possible find the limit.
  1. \(\sumz \left(\frac{2^n-1}{3^n}\right)\): \begin{align*} \sumz \left(\frac{2^n-1}{3^n}\right) &= \sumz (2/3)^n - \sumz (1/3)^n\\[2ex] &= \frac{1}{1-(2/3)} - \frac{1}{1-(1/3)}\\[2ex] &= 3-3/2 = 3/2 \end{align*}
  2. \(\sumz \frac{4^n-7}{4^n}\): \begin{align*} \sumz \frac{4^n-7}{4^n} &= \sumz \left(1-\frac{7}{4^n}\right) \end{align*} Now \(\ds\limi (1-7/4^n) = 1\) and thus the series diverges by the Divergence test.
  3. \(\sumz \ln\left(\frac{1}{3^n}\right)\) : We have that \[ \limi \ln\left(\frac{1}{3^n}\right) = -\infty \] and thus the series \(\ds\sumz \ln\left(\frac{1}{3^n}\right)\) diverges by the Divergence test.
  4. \(\sumz \frac{\cos(n\pi)}{5^n}\): Notice that \(\cos(0\pi) = 1\), \(\cos(1\cdot \pi) = -1\), \(\cos(2\cdot\pi) = 1\), and \(\cos(3\cdot \pi) = -1\). Hence, \(\cos(n\pi) = (-1)^n\). Therefore \begin{align*} \sumz \frac{\cos(n\pi)}{5^n} &= \sumz \frac{(-1)^n}{5^n} \\[2ex] &=\sumz \left(\frac{-1}{5}\right)^n \\[2ex] &=\frac{1}{1-(-1/5)} = \frac{5}{6} \end{align*}
  5. \(\sumz e^{-2n}\): First notice that \[ \limi e^{-2n} = 0 \] and thus the series may converge. Now \[ e^{-2n} = (e^{-2})^n = \left(\frac{1}{e^2}\right)^n \] Since \(|1/e^2| \lt 1\) then the geometric series converges to \[ \sumz e^{-2n} = \sumz (e^{-2})^n = \frac{1}{1-e^{-2}} = \frac{e^2}{e^2-1} \]
  6. \(\sum_{n=3}^\infty \left(\frac{5}{6}\right)^n\): This resembles a geometric series except the initial value of the index is \(n=3\). However, we can write \begin{align*} \sum_{n=3}^\infty \left(\frac{5}{6}\right)^n = \sumz \left(\frac{5}{6}\right)^{n+3} &= \sumz \left(\frac{5}{6}\right)^{n}\left(\frac{5}{6}\right)^{3}\\[2ex] &=\left(\frac{5}{6}\right)^{3} \sumz \left(\frac{5}{6}\right)^{n}\\[2ex] &=\left(\frac{5}{6}\right)^{3} \cdot \frac{1}{1-(5/6)}\\ &=\left(\frac{5}{6}\right)^{3} \cdot 6 \end{align*} An alternative method is to expand the series: \begin{align*} \ds\sum_{n=3}^\infty \left(\frac{5}{6}\right)^n &= (5/6)^3 + (5/6)^4 + (5/6)^5 + (5/6)^6+ \cdots\\[2ex] &= (5/6)^3 \left(1 + (5/6) + (5/6)^2 + (5/6)^3 + \cdots \right)\\[2ex] &= (5/6)^3 \sumz (5/6)^n \end{align*}
A particular series \(\sumo a_n\) has sequence of partial sums \((s_n)\) whose \(n\)th general term is \(\ds s_n = \frac{3n+1}{n}\). (5 minute group work, 3 students give answer on board)
  1. Find \(\ds\sum_{k=1}^5 a_k\) and \(\ds\sum_{k=1}^{10} a_k\).
  2. Does the series \(\sumo a_n\) converge? Explain. If yes, what does it converge to?
  3. Does the sequence \((a_n)\) converge? Explain. If yes, what does it converge to?
One last note: \[ \sumo a_n b_n \neq \sumo a_n \sumo b_n. \]

Integral Test

So far, the only series we have been able to show converges is the Geometric series \(\ds\sumz r^n\) provided \(|r| \lt 1\), and any of its variants. This was possible because we were able to show that \(s_n = \frac{1-r^n}{1-r}\) and then \(\ds\limi s_n = \frac{1}{1-r}\). In the next few sections, we will develop general tests to deduce the convergence of other series. \\ Our first general test is the Integral test and relies on the Monotone Convergence theorem. Recall that given a series \(\sum a_n\) if the sequence of partial sums \((s_n)\) is increasing and bounded above then \((s_n)\) converges by the MCT and thus \(\sum a_n\) converges. We now determine under what circumstances \((s_n)\) is increasing. Recall that by definition \[ s_n = a_1 + a_2 + \cdots + a_n \] and thus \[ s_{n+1} = s_n + a_{n+1} \] If the sequence \((a_n)\) has all non-negative terms, that is, \(a_n \geq 0\) for all \(n\), then \[ s_n \leq s_{n+1} \] and thus \((s_n)\) is increasing. Thus, in this case, \((s_n)\) converges if it is bounded above.\\ Consider the series \(\sumo \frac{1}{n^2}\). Since \(a_n = \frac{1}{n^2} \geq 0\) then \((s_n)\) is increasing. If \((s_n)\) is bounded above then \(\sumo \frac{1}{n^2}\) converges. We can view the series as an infinite Riemann sum:
figures/integral-test.svg
Compare the area under the function \(f(x)=\frac{1}{x^2}\) from \(x=1\) to \(x=n\): \[ s_n = 1+\frac{1}{2^2} + \frac{1}{3^2} + \cdots + \frac{1}{n^2} \lt 1 + \int_1^n \frac{1}{x^2}\,dx \] But \(\int_1^n \frac{1}{x^2}\,dx \lt \int_1^\infty \frac{1}{x^2}\,dx\) and therefore \[ s_n \lt 1 + \int_1^\infty \frac{1}{x^2}\,dx. \] The improper integral involved converges \begin{align*} \int_1^\infty \frac{1}{x^2}\,dx &= \lim_{t\rightarrow\infty} \int_1^t\frac{1}{x^2}\,dx\\[2ex] &= \lim_{t\rightarrow\infty} \left(-\frac{1}{t}+1\right)\\[2ex] &= 1 \end{align*} Hence, \[ s_n \lt 1 + 1 = 2. \] Therefore, \((s_n)\) is bounded above! By the Monotone Convergence theorem, the series \(\sumo \frac{1}{n^2}\) converges, but we cannot deduce the sum. We write down the general statement of the Integral Test.
Let \((a_n)\) be a sequence with \(a_n\geq 0\) and suppose \(a_n = f(n)\), where \(f\) is non-negative, continuous, and decreasing. Then \(\sum a_n\) is convergent if and only if \(\int_1^\infty f(x)\, dx\) is convergent.
Let \(p \gt 0\) and consider the series \(\sumo \frac{1}{n^p}\). Hence \(a_n = \frac{1}{n^p} = f(n)\) where \(f(x) = \frac{1}{x^p}\). The function \(f\) is continuous, non-negative, and decreasing on the interval \([1,\infty)\). Now \[ \int \frac{1}{x^p}\,dx = \begin{cases} \frac{x^{1-p}}{1-p}, & p\neq 1\\[2ex] \ln|x|, & p=1 \end{cases} \] and then \begin{align*} \int_1^\infty \frac{1}{x^p}\, dx &= \lim_{t\rightarrow\infty} \int_1^t \frac{1}{x^p}\, dx \\[2ex] &= \begin{cases} \ds\lim_{t\rightarrow\infty} \left(\frac{t^{1-p}}{1-p} - \frac{1}{1-p}\right), & p\neq 1\\[2ex] \ds\lim_{t\rightarrow\infty} \ln|t| , & p=1 \end{cases}\\[2ex] &= \begin{cases} \frac{1}{p-1}, & p \gt 1\\[2ex] \text{DNE}, & \text{otherwise} \end{cases} \end{align*} Therefore, the general \(p\)-series \(\sumo \frac{1}{n^p}\) converges if \(p \gt 1\) and diverges otherwise. For example, \(\sumo \frac{1}{n^{1.1}}\), \(\sumo \frac{1}{n^{5}}\) converge but \(\sumo\frac{1}{n^{0.7}}\) diverges.
Determine if the given series converge or diverge.
  1. \(\sumo \frac{1}{1+n^2}\): In this case, \(a_n = f(n)\) where \(f(x) = \frac{1}{1+x^2}\), and \(f\) is continuous, decreasing, and non-negative on \([1,\infty)\). Now \begin{align*} \int_1^\infty \frac{1}{1+x^2}\,dx &= \lim_{t\rightarrow\infty} (\arctan(t) - \arctan(1))\\ &=\frac{\pi}{2} - \frac{\pi}{4}=\frac{\pi}{4}. \end{align*} Therefore, by the integral test, the given series converges because the improper integral converges.
  2. \(\sumo ne^{-n^2}\): First notice that \(\limi \frac{n}{e^{n^2}} = 0\) and therefore the series may converge. In this case, \(a_n = f(n)\) where \(f(x) = x e^{-x^2}\), and \(f\) is continuous and non-negative on the interval \([1,\infty)\). Now \[ f'(x) = e^{-x^2} - 2x^2 e^{-x^2} = e^{-x^2}(1-2x^2) \lt 0 \] when \(x\geq 1\) and thus \(f\) is decreasing on \([1,\infty)\). Now \begin{align*} \int_1^\infty xe^{-x^2}\, dx &= \lim_{t\rightarrow\infty} \left( -\frac{1}{2}e^{-x^2}\right)\Big|_1^t \\[2ex] &= \lim_{t\rightarrow\infty} \left(-\frac{1}{2}e^{-t^2} + \frac{1}{2}e^{-1}\right)\\[2ex] &= \frac{1}{2}e^{-1} \end{align*} Therefore, since the improper integral converges, then by the Integral test the given series converges.
  3. \(\sumo \frac{\ln(n)}{n}\): First notice that \[ \limi \frac{\ln(n)}{n} = \limi \frac{1/n}{1} = 0 \] and thus the sequence may converge. Now \(a_n = f(n)\) where \(f(x) = \frac{\ln(x)}{x}\), and \(f\) is continuous, non-negative, and decreasing on \([1,\infty)\). Now \begin{align*} \int_1^\infty \frac{\ln(x)}{x} \, dx &= \lim_{t\rightarrow\infty} \frac{1}{2}(\ln(x))^2\Big|_1^t \\[2ex] &= \lim_{t\rightarrow\infty} \frac{1}{2} (\ln(t))^2\\ &= \infty \end{align*} Hence, since the improper integral diverges then the given series diverges.

Comparison Tests

Thus far, we know that
  1. \(\sumz r^n = \frac{1}{1-r}\) if \(|r| \lt 1\),
  2. \(\sumo \frac{1}{n}\) diverges, and
  3. \(\sumo \frac{1}{n^p}\) converges only if \(p \gt 1\).
We now introduce Comparison tests which allow us to deduce either convergence/divergence of a given series \(\sum a_n\) when we know the convergence/divergence of another series \(\sum b_n\) and when \(a_n \leq b_n\).
Suppose that \((a_n)\) and \((b_n)\) are sequences of non-negative terms: \(a_n\geq 0\) and \(b_n\geq 0\). Suppose further that \(a_n \leq b_n\) for all \(n\). The following hold:
  1. If \(\sum b_n\) converges then \(\sum a_n\) converges.
  2. If \(\sum a_n\) diverges then \(\sum b_n\) diverges.
Notice that this theorem does not say what happens when \(a_n\leq b_n\) and when \(\sum a_n\) converges or \(\sum b_n\) diverges. In what follows, the following fact will be used frequently. If \(a, b \gt 0\) and \(a \lt b\) then \(\frac{1}{b} \lt \frac{1}{a}\): \[ 0 \lt a \lt b \;\Longrightarrow \; \frac{1}{b} \lt \frac{1}{a}. \] Also, if \(c \gt 0\) then also \[ a c \lt b c \qquad \text{and} \frac{a}{c} \lt \frac{b}{c}. \]
\(\ds\sumo \frac{5}{5n-1}\)
First \(\ds\limi \frac{5}{5n-1} = 0\), hence Divergence test is inconclusive. Also, \(a_n \geq 0\) for all \(n\). When \(n\) is very large, \(5n-1\approx 5n\) and thus \[ \frac{5}{5n-1} \approx \frac{5}{5n} = \frac{1}{n}. \] Hence, when \(n\) is large the terms of the series are like the terms of the Harmonic series. We have reason to believe then that the given series diverges. Now because \(5n-1 \lt 5n\) then \(\frac{1}{5n} \lt \frac{1}{5n-1}\) and thus \[ \frac{5}{5n} \lt \frac{5}{5n-1}. \] The series \(\sumo \frac{5}{5n} = \sumo \frac{1}{n}\) diverges and thus by the DCT the given series diverges.
\(\ds\sumo \frac{n+3}{n^4+5}\)
We have that \(a_n = \frac{n+3}{2n^4+5}\geq 0\) and \((a_n)\rightarrow 0\). When \(n\) is large \[ a_n = \frac{n+3}{2n^4+5} \approx \frac{n}{2n^4} = \frac{1}{2n^3}. \] Thus, the terms of the series behave like the terms of the series \(\sumo \frac{1}{2n^3}\); we have reason to believe that the given series is convergent. Now, \[ \frac{n+3}{2n^4+5} \lt \frac{n+3}{2n^4} \leq \frac{n+3n}{2n^4} = \frac{2}{n^3}. \] Hence, \(a_n \leq \frac{2}{n^3}\). The series \(\sum \frac{2}{n^3}\) converges and thus by the DCT, the given series converges also.
\(\ds\sumo \frac{\ln(n)}{n^2}\)
Here \(a_n = \frac{\ln(n)}{n^2}\). The following is true: \[ \ln(x) \leq \sqrt{x} = x^{0.5} \] and in fact \[ \ln(x) \leq x^{0.4}. \] Below is a graph:
figures/log-sqrt.svg
Therefore, \[ \frac{\ln(n)}{n^2} \leq \frac{n^{0.4}}{n^2} = \frac{1}{n^{1.6}} \] The \(p\)-series \(\sumo \frac{1}{n^{1.6}}\) converges and thus by the DCT test the given series converges.
\(\ds\sum_{n=2}^\infty \frac{n+2}{n^2-n}\)
Here \(a_n = \frac{n+2}{n^2-n}\geq 0\) for all \(n\geq 2\), and when \(n\) is large \[ a_n = \frac{n+2}{n^2-n} \approx \frac{n}{n^2} = \frac{1}{n}. \] Thus, there is reason to believe that the series diverges. Now \[ a_n = \frac{n+2}{n^2-n} \geq \frac{n+2}{n^2} \gt \frac{n}{n^2} = \frac{1}{n}. \] Thus, \(\frac{1}{n} \leq a_n\). Since the series \(\sum_{n=2}^\infty \frac{1}{n}\) diverges then by the DCT test, the given series diverges.
We see that applying the Direct Comparison test requires some ingenuity. The following theorem is derived from the Direct Comparison test and some cases is easier to apply than the DCT.
Suppose that \(a_n \gt 0\) and \(b_n \gt 0\) for all \(n\).
  1. If \(\ds\limi \frac{a_n}{b_n} = c\) exists and \(c \gt 0\) then \(\sum a_n\) and \(\sum b_n\) have the same convergence/divergence property, i.e., either both converge or both diverge.
  2. If \(\ds\limi \frac{a_n}{b_n} = 0\) and if \(\sum b_n\) converges then \(\sum a_n\) converges also.
  3. If \(\ds\limi \frac{a_n}{b_n} = \infty\) and if \(\sum b_n\) diverges then \(\sum a_n\) also diverges.
Usually, we know the convergence/divergence properties of \(\sum b_n\) and we are interested in knowing the convergence/divergence properties of \(\sum a_n\). Thus we compute the limit \[ \limi \frac{a_n}{b_n} \] and apply the LCT. However, the difficult part with the LCT is to first determine the correct series \(\sum b_n\) to compare with \(\sum a_n\) and this is done using the preliminary analysis. Let's redo the previous examples with the LCT.
\(\ds\sumo \frac{5}{5n-1}\)
As before, we \(n\) is large \[ a_n = \frac{5}{5n-1} \approx \frac{5}{5n} = \frac{1}{n}. \] Now, \(\sumo b_n = \sumo \frac{1}{n}\) diverges and \[ \limi \frac{a_n}{b_n} = \limi \frac{5n}{5n-1} = 1 \gt 0. \] Hence, by the LCT, the given series diverges.
\(\ds\sumo \frac{n+3}{2n^4+5}\)
As before, when \(n\) is large \[ a_n = \frac{n+3}{2n^4+5} \approx \frac{n}{2n^4} = \frac{1}{2n^3}. \] The series \(\sumo b_n = \sumo \frac{1}{n^3}\) converges and \[ \limi \frac{a_n}{b_n} = \limi \frac{n^4 + 3n^3}{2n^4 + 5} = \frac{1}{2} \gt 0. \] Thus, by the LCT, the given series converges. Suppose we compare with \(\sumo b_n = \sumo \frac{1}{n^2}\), which converges. Now \[ \limi \frac{a_n}{b^n} = \limi \frac{n^3+3n^2}{2n^4 + 5} = 0. \] Then since \(\sumo b_n\) converges then the given series also converges.
\(\ds\sumo \frac{\ln(n)}{n^2}\)
Suppose we blindly compare with \(\sumo b_n = \sumo \frac{1}{n}\), which diverges: \[ \limi \frac{a_n}{b_n} = \limi \frac{n \ln(n)}{n^2} = \limi \frac{\ln(n)}{n} = 0. \] When the limit is zero, we can only conclude that \(\sumo a_n\) converges if \(\sumo b_n\) converges; but \(\sumo b_n\) does not converge and thus the test is inconclusive using \(\sumo \frac{1}{n}\). If we try with \(\sumo \frac{1}{n^2}\), which converges, we get \[ \limi \frac{a_n}{b_n} = \limi \ln(n) = \infty. \] In this case, part (iii) of the LCT is not applicable because \(\sumo \frac{1}{n^2}\) converges. In this case, the DCT is actually easier because we know the bound \(\ln(n) \leq \sqrt{n}\) and then \[ \frac{\ln(n)}{n^2} \leq \frac{n^{0.5}}{n^2} = \frac{1}{n^{1.5}} \] and \(\sumo \frac{1}{n^{1.5}}\) converges. Note that if we try the LCT with \(\sumo b_n = \sumo \frac{1}{n^{1.5}}\) which converges then \begin{align*} \limi \frac{a_n}{b_n} = \limi \frac{n^{1.5}\ln(n)}{n^2} \limi \frac{\ln(n)}{n^{0.5}} &= \limi \frac{(1/n)}{\frac{1}{2}n^{-1/2}} \\ &=\limi \frac{2}{n^{1/2}} = 0 \end{align*} Hence, part (ii) of the LCT is applicable and we conclude that the given series is convergent.
\(\ds\sumo \frac{1-n}{n2^n}\)
Here \(a_n = \frac{1-n}{n2^n}\) is negative for \(n \gt 1\) and thus cannot apply the comparison tests directly. However, notice that \[ \sumo \frac{1-n}{n2^n} = \sumo \frac{1}{n2^n} - \sumo \frac{1}{2^n} \] The second series is geometric with \(|r|=1/2 \lt 1\) and thus converges. For the first series we have \[ \frac{1}{n2^n} \leq \frac{1}{2^n} \] and thus by the D.C. the series \(\sumo \frac{1}{n2^n}\) converges. Hence, the given series converges.
\(\ds\sumo \frac{n^2}{\sqrt{n^9+2}}\)
Notice that \[ \frac{n^2}{\sqrt{n^9+2}} \leq \frac{n^2}{\sqrt{n^9}} = \frac{1}{n^{2.5}} \] The \(p\)-series \(\sumo \frac{1}{n^{2.5}}\) converges and thus by the D.C. the given series converges.
\(\ds\sumo \frac{7n^2}{\sqrt[3]{n^9+2}}\)
When \(n\) is large \[ \frac{n^2}{\sqrt[3]{n^9+2}} \approx \frac{7n^2}{n^3} = \frac{7}{n} \] and so we have reason to believe the series diverges. Now \[ \frac{n^2}{\sqrt[3]{n^9+2}} \geq \frac{n^2}{\sqrt[3]{n^9+7n^9}} = \frac{n^2}{\sqrt[3]{8n^9}}=\frac{1}{2n} \] Since \(\sum \frac{1}{2n}\) diverges then the given series diverges. Alternatively, could use limit comparison test with \(b_n=\frac{1}{n}\).

Absolute Convergence and Ratio Test

Consider the series \[\ds\sumo \frac{(-1)^n}{n^2}.\] This is called an alternating series. The series is clearly not geometric, we cannot apply integral test or any comparison test because the series contains negative terms. However, we know that \[ \sumo |a_n| = \sumo \frac{1}{n^2} \] converges. Does this imply that the original series converges? Well \[ -|a_n| \leq a_n \leq |a_n| \] and thus \[ 0 \leq a_n + |a_n| \leq 2|a_n| \] If \(\sum |a_n|\) converges then \(\sum(a_n +|a_n|)\) converges by the Direct Comparison test. Therefore, by the series laws \[ \sum (a_n + |a_n|) - \sum |a_n| = \sum a_n \] converges also.
Let \(\sum a_n\) be any given series. If \(\sum |a_n|\) converges then \(\sum a_n\) converges. In this case, we say that \(\sum a_n\) converges absolutely.
Because \(\sum |a_n|\) has non-negative terms, we can apply the integral test or any of the comparison tests to \(\sum |a_n|\), when they are applicable.
We can then conclude that the series \(\sumo \frac{(-1)^n}{n^2}\) converges absolutely because \[ \sumo |a_n| = \sumo \frac{1}{n^2} \] is a convergent \(p\)-series.
Consider the series \(\ds\sumo \frac{n^4}{(-3)^n}\). Here we have \[ \sumo |a_n| = \sumo \frac{n^4}{3^n}. \] Applying L.H. rule, we get \[ \limi |a_n| = \limi \frac{4!}{3^n (\ln(3))^4} = 0 \] The exponential \(3^n\) grows much faster than \(n^4\), so suspect series converges. Let \(b_n=\frac{1}{n^2}\) then \[ \limi \frac{|a_n|}{b_n} = \limi \frac{n^6}{3^n} = 0 \] by applying L.H. Rule. Hence, since \(\sumo \frac{1}{n^2}\) converges then \(\sum |a_n|\) converges. And then by Absolute convergence test, the given series \(\sum a_n\) converges absolutely. Note that \(b_n = \frac{1}{2^n}\) would have also worked.
\(\ds\sumo \frac{(-1)^n (3n+4)}{7n^3+3n-1}\)
Have negative terms so consider \[ \sumo |a_n| = \sumo \frac{3n+4}{7n^3+3n-1} \] Then with \(b_n=\frac{1}{n^2}\) we obtain \[ \limi \frac{|a_n|}{b_n} = \limi \frac{3n^3+4n^2}{7n^3+3n-1} = \frac{3}{7} \lt 1 \] Hence, by LCT, the series \(\sumo |a_n|\) converges because \(\sum b_n\) converges. Then by ACT, the original series \(\sum a_n\) converges absolutely.
\(\ds\sumo \frac{\cos(n)}{n^2}\)
Have negative terms so consider \[ \sumo |a_n| = \sumo \frac{|\cos(n)|}{n^2} \] Suspect converges but if we try the LCT with say \(b_n=\frac{1}{n^2}\) we get \[ \limi \frac{|a_n|}{b_n} = \limi |\cos(n)| \] and this limit DNE. Thus, the LCT is inconclusive. However, notice that \[ \frac{|\cos(n)|}{n^2} \leq \frac{1}{n^2} \] and \(\sum \frac{1}{n^2}\) converges and thus \(\sum |a_n|\) converges by the DCT. Thus, the given series \(\sum a_n\) converges absolutely.
We now introduce a very powerful convergence test.
Let \(\sum a_n\) be any given series with \(a_n \neq 0\) and let \[ \rho = \limi \left| \frac{a_{n+1}}{a_n} \right|. \]
  1. If \(\rho \lt 1\) then \(\sum a_n\) converges absolutely.
  2. If \(\rho \gt 1\) or \(\rho=\infty\) then \(\sum a_n\) diverges.
  3. If \(\rho=1\) or the limit does not exist then the test is inconclusive.
Consider again \(\ds\sumo \frac{n^4}{(-4)^n}\). We compute \begin{align*} \frac{a_{n+1}}{a_n} = \frac{(n+1)^4}{(-4)^{n+1}} \times \frac{(-4)^n}{n^4} = \frac{(n+1)^4}{-4n^4} \end{align*} and then \begin{align*} \limi \left| \frac{a_{n+1}}{a_n} \right| = \limi \frac{(n+1)^4}{4n^4} = \frac{1}{4} \lt 1 \end{align*} and thus by the Ratio Test, the series converges absolutely.
\(\ds\sumo \frac{10^n}{n!}\)
Compute \[ \frac{a_{n+1}}{a_n} = \frac{10^{n+1}}{(n+1)!} \times \frac{n!}{10^n} = \frac{10}{n+1} \] and thus \[ \limi \left| \frac{a_{n+1}}{a_n} \right| = \limi \frac{10}{n+1}=0 = \rho \lt 1. \] Hence, the series converges by the RT.
\(\ds\sumo \frac{(-1)^n n!}{e^n}\)
Compute \begin{align*} \frac{a_{n+1}}{a_n} = \frac{(-1)^{n+1} (n+1)!}{e^{n+1}} \frac{e^n}{(-1)^n n!}=\frac{(-1) (n+1)}{e} \end{align*} and thus \[ \limi \left| \frac{a_{n+1}}{a_n} \right| = \limi \frac{(n+1)}{e} = \infty \] and thus by the RT the given series diverges.
\(\ds\sumo \frac{4^n n!}{(2n)!}\)
We compute \begin{align*} \frac{a_{n+1}}{a_n} = \frac{4^{n+1}(n+1)!}{(2n+2)!} \frac{(2n)!}{4^n n!} = \frac{4 (n+1)}{(2n+2)(2n+1)} \end{align*} and therefore \[ \limi \left| \frac{a_{n+1}}{a_n} \right| = \limi \frac{4 (n+1)}{(2n+2)(2n+1)} = 0 \lt 1 \] and thus by the RT the given series converges.
\(\ds\sumo \frac{(2n)!}{n!n!}\)
We compute \begin{align*} \frac{a_{n+1}}{a_n} = \frac{(2n+2)!}{(n+1)!(n+1)!} \frac{n!n!}{(2n)!} = \frac{(2n+2)(2n+1)}{(n+1)(n+1)} \end{align*} and thus \[ \limi \left| \frac{a_{n+1}}{a_n} \right| = \limi \frac{(2n+2)(2n+1)}{(n+1)(n+1)} = 4 \gt 1 \] and thus by the RT the given series diverges.
The Ratio Test doesn't always work or can be difficult to apply. Consider for instance \[ \sumo \frac{\cos(n)}{n^2+1}  \text{and}  \sumo \frac{(2n+3)(-1)^n}{5n^3+n-2} \] In the first case, we get \[ \limi \left| \frac{a_{n+1}}{a_n} \right| = \limi \frac{|\cos(n+1)| (n^2+1)}{|\cos(n)| ((n+1)^2+1)} \] and the limit DNE. Here, it is easier to apply absolute convergence combined with the DCT. In the second case, the limit computation is just tedious but does give \(\limi \left| \frac{a_{n+1}}{a_n} \right|=0\). Again, could just apply ACT combined with DCT.

Alternating Series

Consider the series \[ \sumo \frac{(-1)^{n+1}}{n}. \] The ratio test is inconclusive because \[ \limi \left| \frac{a_{n+1}}{a_n} \right| = 1 \] and the series of the absolute values \(\sum \frac{1}{n}\) is divergent and so the ACT is inconclusive. The given series is called the alternating Harmonic series. It turns out that this series converges!\\
The alternating series \[ \sumo (-1)^{n+1} u_n \] converges if the following hold:
  1. \(u_n \gt 0\) for all \(n\)
  2. \((u_n)\) is decreasing or eventually decreasing
  3. \(\ds\limi u_n = 0\)
For the alternating series, \(\sumo (-1)^{n+1} \tfrac{1}{n}\), we have \(u_n = \frac{1}{n}\) and all three conditions are satisfied and thus the alternating harmonic series converges. However, because \(\sum |a_n| = \sum \frac{1}{n}\) diverges we also say that \(\sum \frac{(-1)^{n+1}}{n}\) converges conditionally.
\(\sumo \frac{(-1)^{n+1} 10n}{n^2+16}\)
Trying the ratio test gives \[ \limi \left| \frac{a_{n+1}}{a_n} \right| = 1 \] and the series of absolute values \(\sum |a_n| = \sum \frac{10n}{n^2+16}\) diverges, so the ACT is inconclusive. In this case, \[ u_n = \frac{10n}{n^2+16} \gt 0 \] and \(\limi u_n = 0\). Now \[ (u_n) = \left(\tfrac{10}{17}, \tfrac{20}{20}, \tfrac{30}{25}, \tfrac{40}{32}, \tfrac{50}{41}, \tfrac{60}{52}, \tfrac{70}{65}\ldots \right) \] we get that \(u_4 \lt u_5 \lt u_6 \lt \cdots\). To show this, let \(f(x) = \frac{10x}{x^2+16}\). Then \[ f'(x) = \frac{10(x^2+16)-20x^2}{(x^2+16)^2}=\frac{160-10x^2}{(x^2+16)^2} = \frac{10(16-x^2)}{(x^2+16)^2} \] Now \(f'(x) \lt 0\) if and only if \(x \gt 4\) and \(x \lt -4\). Hence, \((u_n)\) is decreasing starting with \(n\geq 4\). All three conditions are met and thus by the Alternating series test the given series converges conditionally.
\(\sumo \frac{(-1)^{n+1} n^2}{n^3+1}\)
Again, the RT test and ACT are inconclusive for the same reasons as the previous example. In this case, \(u_n = \frac{n^2}{n^3+1} \lt 0\) and clearly \(\limi u_n = 0\). Now let \(f(x) = \frac{x^2}{x^3+1}\). Then \[ f'(x) = \frac{x(2-x^3}{(x^3+1)^2} \lt 0 \] iff \(x\geq 2^{1/3}\). So, \(u_n\) decreasing for \(n\geq 2\). Thus, the series converges conditionally.

Power Series (Part I)

Recall that \(\sumz r^n = \frac{1}{1-r}\) provided \(r\in (-1,1)\). Think of \(r\) as a variable that varies in the interval \((-1,1)\), and call it \(x\) instead: \[ \frac{1}{1-x} = \sumz x^n = 1 + x + x^2 + x^3 + x^4 + \cdots \] Hence, in the interval \((-1,1)\), the function \(f(x) = \frac{1}{1-x}\) can be represented as an infinite series. So what? If we can represent \(f(x)=\frac{1}{1-x}\) in this way maybe we can do the same for other more complicated functions. Notice the following: \[ \int \frac{1}{1-u} dx = - \ln|1-u| \] Does this mean then that \begin{align*} -\ln|1-u| &= \int (1+u+u^2+u^3+u^4+\cdots)\, du \\[2ex] &= c + u + \frac{u^2}{x} + \frac{u^3}{3} + \frac{u^4}{4} + \cdots ? \end{align*} Not clear that this is valid. What is \(c\)? At \(u=0\) we have \(\ln|1-0| = 0\) and thus \(c=0\). Hence \[ \ln|1-u| = -u - \frac{u^2}{2} - \frac{u^3}{3} - \frac{u^4}{4} - \cdots \] Therefore, if \(u=1-x\) then \[ \ln|x| = (x-1) - \frac{(x-1)^2}{2} + \frac{(x-1)^3}{3} - \frac{(x-1)^4}{4} + \cdots \] Notice then that \[ \ln(2) = 1 - \frac{1}{2} + \frac{1}{3}-\frac{1}{4}+\cdots = \sumo \frac{(-1)^{n+1}}{n} \] What is the main idea?
A power series is a series of the form \[ c_0 + c_1 (x-a) + c_2 (x-a)^2 + c_3 (x-a)^3 + c_4(x-a)^4 + \cdots \] where the number \(a\) is called the center of the power series and the terms of the sequence \((c_n)=(c_0,c_1,c_2,\ldots)\) are called the coefficients of the power series. Using summation notation: \[ \sumz c_n (x-a)^n = c_0 + c_1 (x-a) + c_2 (x-a)^2 + c_3 (x-a)^3 + \cdots \] When the center is \(a=0\) then \[ \sumz c_n x^n = \sumz c_n x^n = c_0 + c_1x + c_2 x^2 + c_3x^3 + \cdots \]
Expand the given power series by writing out the first few terms of the series.
  1. \(\ds\sumz \frac{n^2}{n!} x^n\)
  2. \(\ds\sumo \frac{1}{2n+1} (x+3)^n\)
Use summation notation to write the given power series.
  1. \(\ds1 - \frac{x}{4} + \frac{x^2}{9} - \frac{x^3}{16} + \frac{x^4}{25} - \cdots = \sumz \frac{(-1)^n x^n}{(n+1)^2}\)
  2. \(\ds-\frac{x}{2} + \frac{x^3}{6} - \frac{x^5}{10} - \frac{x^7}{14} + \cdots = \sumz \frac{(-1)^n x^{2n-1}}{2(2n-1)} \)
  3. \(\ds 1 + \frac{(x+1)^2}{2} + \frac{(x+1)^4}{4} + \frac{(x+1)^6}{8} +\cdots = \sumz \frac{(x+1)^{2n}}{2^n}\)
Given a power series, the first question we would like to answer is for which values of \(x\) does the series converge.
Find all the values of \(x\) for which the given power series converges. \[ \sumo \frac{(-1)^n}{n3^n}(x-1)^n \]
Here the center is \(a=1\). Suppose we set \(x=3\): \[ \sumo \frac{(-1)^n 2^n}{n3^n} \] Apply the Ratio test: \begin{align*} \limi \left| \frac{a_{n+1}}{a_n} \right| &= \frac{2}{3} \lt 1 \end{align*} Thus, when \(x=3\), the resulting series converges. If instead we set \(x=-3\): \[ \sumo \frac{n(-1)^n (-4)^n}{3^n} \] and in this case the Ratio test gives \[ \limi \left| \frac{a_{n+1}}{a_n} \right| = \frac{4}{3} \gt 1 \] and thus the resulting series diverges when \(x=-3\). Apply the Ratio test for general \(x\): \begin{align*} \limi \left| \frac{a_{n+1}}{a_n}\right| &= \limi \left| \frac{(-1)^{n+1}(x-1)^{n+1}}{(n+1)3^{n+1}} \cdot \frac{3^nn}{(-1)^n (x-1)^n}\right|\\ &= |x-1| \limi \frac{n}{3(n+1)}\\ &= |x-1| \cdot\frac{1}{3} = \rho \end{align*} By the Ratio test, the series converges when \(\rho \lt 1\) or when \(|x-1| \lt 3\), or when \(x \in (1-3, 1 +3)=(-2,4)\). Note that the R.T. is inconclusive when \(\rho =1\) and when \(\rho \gt 1\) the series diverges. The case \(\rho=1\) occurs when \(|x-1| = 3\) which happens at the boundary points \(x=-2\) and \(x=4\). We need to check these manually. At \(x=-2\) we obtain \[ \sumo \frac{(-1)^n (-3)^n}{n3^n} = \sumo \frac{1}{n} \] which diverges. On the other hand, at the boundary point \(x=4\) we obtain \[ \sumo \frac{(-1)^n 3^n}{n3^n} = \sumo \frac{(-1)^n}{n} \] which converges. Hence, the interval of convergence is \((-2,4]\). We can therefore define the function \[ f(x) = \sumo \frac{(-1)^n}{n3^n}(x-1)^n \] whose domain is \((-2,4]\).
Find the values of \(x\) where the given power series converges. \[ \sumz \frac{(x+2)^n}{10^n} \]
Applying the Ratio test yields \[ \limi \left|\frac{a_{n+1}}{a_n}\right| = \frac{|x+2|}{10} = \rho \] Hence, by the Ratio test, the series converges if \(|x+2| \lt 10\) or when \(x\in (-12,8)\). When \(\rho=1\), the Ratio test is inconclusive, which occurs when \(x=-12\) or \(x=8\). At \(x=-12\) we obtain \(\sumz (-1)^n\) which diverges and at \(x=8\) we obtain \(\sumz 1\) which also diverges. Hence, the interval where the series converges is \((-12,8)\).
For any given power series \(\sumz c_n (x-a)^n\), one of the following will hold:
  1. The series converges for \(x\) satisfying \(|x-a| \lt R\) where \(R \gt 0\) is a constant, that is, when \(x \in (a-R, a+R)\). At the boundary points \(a\pm R\), the series may or may not converge. Outside the interval, that is, when \(|x-a| \gt R\), the series diverges.
  2. The series converges for any \(x\), that is, the interval of convergence is \((-\infty, \infty)\).
  3. The series converges only for \(x=a\).
The number \(R\) is called the radius of convergence and the interval \((a-R, a+R)\) (with possibly one or both of the boundary points included) is called the interval of convergence. In case (ii), we could say that \(R=\infty\) and in case (iii) that \(R=0\).
For each power series, determine the the interval and radius of convergence.
  1. \(\ds\sumz \frac{(-1)^n}{n!} x^n\qquad\) \((-\infty,\infty)\)
  2. \(\ds\sumz \frac{n!}{n^2} (x-2)^n\qquad\) Only for \(x=2\), \(R=0\)
  3. \(\ds\sumo \frac{4^n x^{2n}}{n}\qquad\) For \(|x| \lt \frac{1}{2}\), \(R=1/2\)
  4. \(\ds\sumo \frac{(3x+7)^n}{4^n n}\qquad\) For \([-11/3, -1)\), \(R=4/3\). Graph the resulting function on the domain \([-11/3,-1)\).
Find the interval of convergence of \[ \sumz \frac{(-1)^{n+1} x^{2n+1}}{(2n+1)!} \] and graph the resulting function on its domain.

Power Series (Part II)

We know that the function \(f(x)=\frac{1}{1-x}\) can be represented as a power series in the interval \((-1,1)\) because \[ \frac{1}{1-x} = \sumz x^n = 1 + x + x^2+x^3 + \cdots \] In this case, \((c_n) = (1,1,1,1,\ldots)\) and the center is \(a=0\). In this section, we are going to use this power series representation of \(f(x) = \frac{1}{1-x}\) to construct power series representations of other functions. Representing a function as a power series has many applications and we will see some applications soon. Suppose that \(f(x)\) is a function defined for \(x\) satisfying \(|x-a| \lt R\), that is, in the interval \((a-R, a+R)\). To give an example, suppose that \(|x-1| \lt 7\) and thus \((a-R,a+R) = (-6,8)\). Let \(u(x) = 2x+3\) and consider the new function \(g(x) = f(u(x)) = f(2x+3)\). What is the domain of \(g\)? The function \(g(x) = f(u(x))\) is well-defined if \begin{align*} |u(x) - 1| \lt 7 &\Longrightarrow |2x+3 - 1| \lt 7 \\ &\Longrightarrow |2x+2| \lt 7\\ &\Longrightarrow |x+1| \lt \tfrac{7}{2} \end{align*} Hence, \(g(x)\) is defined when \(|x+1| \lt \tfrac{7}{2}\) or on the interval \((-1-7/2,-1+7/2) = (-9/2,5/2)\).
Determine a power series representation for the given function and determine the interval where the representation is valid. \[ f(x) = \frac{8}{5+2x} \]
We know that \[ \frac{1}{1-u} = \sumz u^n \] and this is valid for \(|u| \lt 1\) or for \(u\in (-1,1)\). Notice that \begin{align*} f(x) = \frac{8}{5} \left(\frac{1}{1+\tfrac{2x}{5}}\right) &= \frac{8}{5}\left(\frac{1}{1-(-\tfrac{2x}{5})}\right)\\[2ex] &=\frac{8}{5} \sumz (-\tfrac{2x}{5})^n\\ &= \sumz \frac{8(-1)^n 2^n x^n}{5^{n+1}} \end{align*} and this is valid for \(|\tfrac{-2x}{5}| \lt 1\) or for \(|x| \lt \frac{5}{2}\).
Find the interval of convergence of the given power series and find a closed-form expression for the function \(f\) given by the power series. \[ f(x) = \sumz (-1)^n (2x+1)^n \]
Notice that \[ f(x) = \sumz (-(2x+1))^n \] This is a geometric series with \(r=-(2x+1)\) and we know a geometric series converges when \(|-(2x+1)| \lt 1\): \[ |-(2x+1)| \lt 1 \Longrightarrow 2|(x+\tfrac{1}{2})| \lt 1 \Longrightarrow |x+\tfrac{1}{2}| \lt \tfrac{1}{2} \] Hence, interval of convergence is \((-1/2 - 1/2, -1/2 + 1/2) =(-1,0)\) and the closed-form expression is \[ f(x) = \frac{1}{1-(-(2x+1))} = \frac{1}{1+2x+1} = \frac{1}{2x+2} \]
Find a power series representation for the given function \(f\) and determine the interval where the representation is valid. \[ f(x) = \frac{x}{1+3x^2} = \sumz c_n x^n \] What is \(c_7\) and what is \(c_8\)?
Notice that \begin{align*} f(x) = x\left(\frac{1}{1-(-3x^2)}\right) &= x \sumz (-3x^2)^n \\[2ex] &= \sumz (-1)^n 3^n x^{2n+1} \end{align*} and the representation is valid when \(|-3x^2| \lt 1\): \[ |-3x^2| \lt 1 \Longrightarrow 3|x^2| \lt 1 \Longrightarrow |x| \lt \sqrt{1/3} \] Hence, the interval of convergence is \((-\sqrt{1/3},\sqrt{1/3})\). The power series we found is of the form \[ f(x) = \frac{x}{1+3x^2} = \sumz (-1)^n 3^n x^{2n+1} \] In our representation, the polynomial \(x^7\) appears when \(n=3\), hence \(c_7 = (-1)^3 3^3 = -27\) and since all powers of \(x\) appearing in the series are odd then \(c_8 = 0\).
How do we differentiate/integrate functions that are represented as power series?
Suppose that \(f(x) = \sumz c_n (x-a)^n\) with interval of convergence \((a-R, a+R)\). Then the function \(f\) is continuous and differentiable on the interval \((a-R, a+R)\). Moreover, to find \(f'(x)\) and \(\int f(x)\ dx\) as a power series we simply differentiate/integrate the power series of \(f\): \begin{align*} f'(x) & = \frac{d}{dx} \sumz c_n (x-a)^n\\ &= \frac{d}{dx}\left(c_0 + c_1(x-a) + c_2(x-a)^2 + c_3(x-a)^3 + c_4 (x-a)^4 + \cdots \right)\\ &= c_1 + 2c_2 (x-a) + 3c_3 (x-a)^2 + 4c_4 (x-a)^3 + \cdots \\ &= \sumo n c_n (x-a)^{n-1} \end{align*} And \begin{align*} \int f(x) \, dx &= \int \sumz c_n (x-a)^n \, dx\\[2ex] &= \int \left(c_0 + c_1(x-a) + c_2(x-a)^2 + c_3(x-a)^3 +\cdots\right)dx\\ &= C + c_0(x-a) + c_1 \frac{(x-a)^2}{2} + c_2 \frac{(x-a)^3}{3} + c_3 \frac{(x-a)^4}{4} + \cdots\\ &= C + \sumz c_n \frac{(x-a)^{n+1}}{(n+1)} \end{align*}
When we integrate, the new power series may now converge at the boundary points, and conversely, when we differentiate we may lose convergence at the boundary points. This must be checked on a case-by-case basis.
Find a power series representation for \(f(x) = \arctan(x)\) and determine where the representation is valid. Use your power series to show that \[ \pi = 4 - \frac{4}{3} + \frac{4}{5} - \frac{4}{7} + \cdots \]
We know that \[ f'(x) = \frac{1}{1+x^2} \] Now \[ \frac{1}{1+x^2} = \sumz (-x^2)^n = \sumz (-1)^n x^{2n} \] and this representation is valid when \(|-x^2| \lt 1\) or \(|x| \lt 1\). Therefore, \begin{align*} \arctan(x) = \int \frac{1}{1+x^2}\, dx &= \int \sumz (-1)^n x^{2n} \, dx \\[2ex] &= C + \sumz (-1)^n \frac{x^{2n+1}}{2n+1} \end{align*} To find \(C\) we evaluate both sides at a convenient point, say \(x=0\) and this gives \(C=0\). Hence, \[ \arctan(x) = \sumz (-1)^n \frac{x^{2n+1}}{2n+1} = x - \frac{x^3}{3} + \frac{x^5}{5} - \frac{x^7}{7} + \cdots \] and this is valid for \((-1,1)\). However, notice that at \(x=1\) we get \[ \sumz \frac{(-1)^n}{2n+1} \] which by the Alternating series test converges. Hence, \[ \arctan(1) = 1 - \frac{1}{3} + \frac{1}{5} - \frac{1}{7} + \cdots \] and since \(\arctan(1) = \frac{\pi}{4}\) we get \[ \frac{\pi}{4} = 1 - \frac{1}{3} + \frac{1}{5} - \frac{1}{7} + \cdots \]
Find a power series representation for \(f(x) = x\ln(2+3x)\) and the interval where the representation is valid.
If we can find a power series representation for \(\ln(2+3x)\) then we will need to multiply the series by \(x\) to obtain a power series for \(f(x)\). Now, \[ \frac{d}{dx} \ln(2+3x) = \frac{3}{2+3x} \] so we will find a power series for \(\frac{3}{2+3x}\) and then integrate to get a power series for \(\ln(2+3x)\). Now \begin{align*} \frac{3}{2+3x} = \frac{3/2}{1+3x/2} = \sumz \frac{3}{2} (-3x/2)^n = \sumz \frac{(-1)^n 3^{n+1} x^n}{2^{n+1}} \end{align*} valid for \(|-3x/2| \lt 1\) or for \(|x| \lt 2/3\). Then \[ \ln(2+3x) = \int \sumz \frac{(-1)^n 3^{n+1} x^n}{2^{n+1}} \, dx = C + \sumz \frac{(-1)^n 3^{n+1} x^{n+1}}{2^{n+1}(n+1)} \] and valid for \(|x| \lt 2/3\). Now, evaluating both sides at \(x=0\) gives \(C=\ln(2)\). Hence \[ x\ln(2+3x) = x \ln(2) + \sumz \frac{(-1)^n 3^{n+1} x^{n+2}}{2^{n+1}(n+1)} \]
Find a power series representation for \(f(x) = \frac{1}{(1+x)^2}\) and where the representation is valid.
We know that \[ \frac{1}{1+x} = \sumz (-x)^n = \sumz (-1)^n x^n \] valid in the interval \((-1,1)\). Now, \[ \frac{d}{dx} \frac{1}{1+x} = \frac{-1}{(1+x)^2} \] Therefore, \[ \frac{-1}{(1+x)^2} = \frac{d}{dx} \sumz (-1)^n x^n = \sumo (-1)^n n x^{n-1} \] and thus \begin{align*} \frac{1}{(1+x)^2} &= \sumo (-1)^{n+1} n x^{n-1} \\[2ex] &= 1 - 2x + 3x^2 - 4x^3 + \cdots \end{align*} and the representation is valid in the interval \((-1,1)\).
Find a power series representation for the given function and where the representation is valid \[ \ds f(x) = \int \frac{1}{1+x^7}\, dx,\qquad f(0)=-1 \] If \(f(x) = \sumz c_n x^n\) what is \(c_3\) and \(c_{22}\)?
Compute directly \begin{align*} f(x) &= \int \frac{1}{1+x^7}\, dx \\[2ex] &= \int \sumz (-x^7)^n \, dx\\[2ex] &= C + \sumz (-1)^n \frac{x^{7n+1}}{7n+1} \end{align*} Since \(f(0)=-1\) then \(C=0\) and the interval is \((-1,1)\). The power \(x^3\) does not appear and thus \(c_3=0\). In our representation, the term \(x^{15}\) corresponds to \(n=3\) and thus \(c_{22} = \frac{-1}{22}\).
Show that \[ \ln|1-x| = -\sumo \frac{x^n}{n} \] and determine where the representation is valid. Use your representation to show that \[ \ln(2) = \sumo \frac{(-1)^{n+1}}{n} = 1 - \frac{1}{2} + \frac{1}{3} - \frac{1}{4} + \cdots \]

Taylor Series

We saw how we could use the geometric series to find power series representations for functions such as \(f(x)=\frac{1}{1+x^2}\), \(f(x) = \arctan(x)\), or \(f(x) = x\ln(1+2x)\). We now study a systematic alternative method to find power series representations for any reasonably behaved function. Suppose that \(f(x)\) is a given function in closed-form and we wish to represent it as a power series: \[ f(x) = \sumz c_n (x-a)^n = c_0 + c_1(x-a) + c_2(x-a)^2 + c_3(x-a)^3 + \cdots \] Because a power series is determined by its coefficients, to find a power series for \(f\) we need to find \(c_0, c_1, c_2, \ldots\). First of all, notice that \[ f(a) = c_0. \] Now consider \[ f'(x) = c_1 + 2c_2(x-a) + 3c_3(x-a)^2 + 4c_4(x-a)^3 + \cdots \] and thus \[ f'(a) = c_1 \] Similarly, \[ f^{(2)}(x) = 2c_2 + 3\cdot 2 c_3 + 4\cdot 3 c_4 (x-a)^2 + 5\cdot 4 c_5 (x-a)^3 + \cdots \] and thus \[ f^{(2)}(a) = 2 c_2\; \Longrightarrow\; c_2 = \frac{f^{(2)}(a)}{2} \] Similarly, \[ f^{(3)}(x) = 3\cdot 2 c_3 + 4\cdot 3\cdot 2 c_4(x-a) + \cdots \] and thus \[ f^{(3)}(a) = 3! c_3\; \Longrightarrow\; c_3 = \frac{f^{(3)}(a)}{3!} \] This pattern continues: \[ c_n = \frac{1}{n!} f^{(n)}(a) \] Thus, if \(f(x)\) is represented as a power series centered at \(x=a\) then it must be \begin{align*} f(x) &= \sumz \frac{f^{(n)}(a)}{n!} (x-a)^n \\[2ex] &=f(a) + f'(a) (x-a) + \frac{f^{(2)}(a)}{2!} (x-a)^2 + \frac{f^{(3)}(a)}{3!}(x-a)^3 + \\ &\qquad \frac{f^{(4)}(a)}{4!} (x-a)^4 + \cdots \end{align*} This power series is called the Taylor series of \(f\) centered at \(a\). If \(a=0\), then the series is called the Maclaurin series, so in this case: \[ f(x) = \sumz \frac{f^{(n)}(0)}{n!} x^n. \]
Recall that we found a power series representation for \(f(x) = \arctan(x)\): \[ f(x) = \arctan(x) = \sumz (-1)^n \frac{x^{2n+1}}{(2n+1)} = x - \frac{x^3}{3} + \frac{x^5}{5} - \frac{x^7}{7} + \cdots \] This was obtained by integrating the power series for \(\frac{1}{1+x^2}\). Let's compute the Maclaurin series of \(f\) to compare: \begin{align*} f(x) &= \arctan(x) & f(0) &= 0 \\[2ex] f'(x) &= \frac{1}{1+x^2} & f'(0) &= 1\\[2ex] f^{(2)} &= \frac{-2x}{(1+x^2)^2} & f^{(2)}(0) &= 0 \\[2ex] f^{(3)} &= \frac{6x^2-2}{(x^2+1)^3} & f^{(3)}(0) &= -2\\[2ex] f^{(4)} &= {\frac {-24\,{x}^{3}+24\,x}{ \left( {x}^{2}+1 \right) ^{4}}} & f^{(4)}(0) &=0 \end{align*} \begin{align*} f^{(5)}(x) &= {\frac {120\,{x}^{4}-240\,{x}^{2}+24}{ \left( {x}^{2}+1 \right) ^{5}}} & f^{(5)}(0) &= 24=4!\\[2ex] f^{(6)}(x) &= {\frac {-720\,{x}^{5}+2400\,{x}^{3}-720\,x}{ \left( {x}^{2}+1 \right) ^{6}}} & f^{(6)}(0) &= 0\\[2ex] f^{(7)}(x) &= 720\,{\frac {7\,{x}^{6}-35\,{x}^{4}+21\,{x}^{2}-1}{ \left( {x}^{2}+1 \right) ^{7}}} & f^{(7)}(0) &= -720 = -6! \end{align*} Therefore, the Maclaurin series is \begin{align*} \sumz \frac{f^{(n)}(0)}{n!} x^n &= x - \frac{2!}{3!} x^3 + \frac{4!}{5!} x^5 - \frac{6!}{7!} x^7 + \cdots \\[2ex] &= x - \frac{1}{3} x^3 + \frac{1}{5} x^5 - \frac{1}{7}x^7 + \cdots \end{align*} which is exactly what we computed when using the geometric series.
Find the Maclaurin series of \(f(x) = \sin(x)\) and determine its interval of convergence. What is \(T_8(x)\), the 8th order Taylor polynomial centered at \(a=0\)?
We compute \begin{align*} f(x) &= \sin(x) & f(0) &= 0 \\ f'(x) &= \cos(x) & f'(0) &= 1 \\ f^{(2)}(x) &= -\sin(x) & f^{(2)}(0) &= 0 \\ f^{(3)}(x) &= -\cos(x) & f^{(3)}(0) &= -1\\ f^{(4)}(x) &= \sin(x) & f^{(4)}(0) &= 0 \\ f^{(5)}(x) &= \cos(x) & f^{(5)}(0) &= 1 \end{align*} and the pattern repeats. We get \begin{align*} \sumz \frac{f^{(n)}(0)}{n!} x^n &= f(0) + f'(0) x + \frac{f^{(2)}(0)}{2!} x^2 + \cdots \\ &= x - \frac{x^3}{3!} + \frac{x^5}{5!} - \frac{x^7}{7!} + \frac{x^9}{9!} - \cdots\\ &= \sumz (-1)^n \frac{x^{2n+1}}{(2n+1)!} \end{align*} Therefore, \[ T_8(x) = x - \frac{x^3}{3!} + \frac{x^5}{5!} - \frac{x^7}{7!} \] Performing the Ratio test on the Maclaurin series we get \[ \limi \left|\frac{a_{n+1}}{a_n}\right| = 0 \] and thus the interval of convergence is \((-\infty,\infty)\). Therefore, \[ \sin(x) = \sumz (-1)^n \frac{x^{2n+1}}{(2n+1)!} \]
Compute the Maclaurin series of \(f(x)=\cos(x)\) and find its interval of convergence.
We could proceed as was done for \(\sin(x)\) or we could use the fact that \(\frac{d}{dx} \sin(x) = \cos(x)\). Differentiating the power series of \(\sin(x)\) we obtain \begin{align*} \cos(x) &= \sumz (-1)^n \frac{x^{2n}}{(2n)!}\\ &= 1 - \frac{x^2}{2!} + \frac{x^4}{4!} - \frac{x^6}{6!} + \cdots \end{align*} with interval of convergence \((-\infty, \infty)\).
Find the Maclaurin series of \(f(x) = e^x\) and determine its interval of convergence. What is \(T_4(x)\), the 4th order Taylor polynomial centered at \(a=0\)?
We need to compute \(f^{(n)}(0)\). But since \(f^{(n)}(x) = e^x\) for all \(n\) then \(f^{(n)}(0) = 1\). Thus the Maclaurin series is \[ \sumz \frac{f^{(n)}(0)}{n!} x^n = 1 + x + \frac{x^2}{2!} + \frac{x^3}{3!} + \frac{x^4}{4!} + \frac{x^5}{5!} + \cdots \] And thus \[ T_4(x) = 1 + x + \frac{x^2}{2!} + \frac{x^3}{3!} + \frac{x^4}{4!} \] To find the interval of convergence compute \[ \limi \left|\frac{a_{n+1}}{a_n}\right| = 0 \] and thus the interval of convergence is \((-\infty, \infty)\). Therefore, \[ e^x = \sumz \frac{x^n}{n!} \] This shows, for instance, that \[ e = 1 + 1 + \frac{1}{2!} + \frac{1}{3!} + \frac{1}{4!} + \cdots \]
Find a power series representation for \(f(x) = e^{2x}\) centered at \(x=1\).
Compute \begin{align*} f(x) &= e^{2x} & f(1) &= e^2\\ f'(x) &= 2e^{2x} & f'(1) &= 2e^2\\ f^{(2)}(x) &= 2^2 e^{2x} & f^{(2)}(1) &= 2^2 e^2\\ f^{(3)}(x) &= 2^3 e^{2x} & f^{(3)}(1) &= 2^3 \end{align*} The pattern is \(f^{(n)}(1) = 2^n e^2\). Therefore, the series centered at \(x=1\) is \[ \sumz \frac{f^{(n)}(1)}{n!} (x-1)^n = \sumz \frac{2^n e^2}{n!} (x-1)^n \]
Suppose that \[ f(x) = \sumz \frac{f^{(n)}}{n!}(x-a)^n = c_0 + c_1(x-a) + c_2(x-a)^2 + c_3(x-a)^3 + \cdots \] and valid on the interval \((a-R, a+R)\). To approximate the infinite sum, we could use finite sum \begin{align*} T_N(x) &= f(a) + f'(a) (x-a) + \frac{f^{(2)}(a)}{2!} (x-a)^2 + \frac{f^{(3)}(a)}{3!}(x-a)^3 \\ &\qquad + \cdots + \frac{f^{(N)}}{N!} (x-a)^N \end{align*} This is called the \(N\)th order Taylor polynomial of \(f\) centered at \(x=a\). When \(x\) is close to the center \(a\), then \[ f(x) \approx T_N(x). \] The higher the \(N\), the more accurate the approximation is.
Consider the function \[ f(x) = \int_0^x e^{t^3}\, dt. \]
  1. Find the Maclaurin series of \(f\) and find its interval of convergence.
  2. Find \(f^{(10)}(0)\).
  3. Find \(T_8(x)\).
  4. Use \(T_8(x)\) to estimate \(f(0.5)\).
Compute \begin{align*} f(x) = \int_0^x e^{t^3}\,dx &= \int_0^x \sumz \frac{1}{n!} t^{3n}\\ &= \sumz \frac{1}{n!} \frac{t^{3n+1}}{(3n+1)}\Big|_0^x\\ &= \sumz \frac{1}{n!} \frac{x^{3n+1}}{(3n+1)} \end{align*} Because the power series of \(e^x\) converges on \((-\infty,\infty)\) then the Taylor series of \(f(x)\) converges on \((-\infty,\infty)\). To find \(f^{(10)}(0)\) recall that a function's power series is its Taylor series, so if \[ f(x) = \sumz c_n (x-a)^n \] then \[ \sumz c_n (x-a)^n = \sumz \frac{f^{(n)}(a)}{n!} (x-a)^n \] Then \[ c_n = \frac{f^{(n)}(a)}{n!} \] and thus \[ f^{(n)}(a) = n! c_n \] Now from our power series representation, the term \(x^{10}\) corresponds to \(n=3\) and thus \[ c_{10} = \frac{1}{3! 10} = \frac{1}{60} \] Then \[ f^{(10)}(0) = 10! \frac{1}{60} \] Now to find \(T_8(x)\) we expand and keep only up to the term \(x^8\) if present: \[ \sumz \frac{1}{n!} \frac{x^{3n+1}}{(3n+1)} = x + \frac{1}{1!\cdot 4} x^4 + \frac{1}{2!\cdot 7} x^7 + \frac{1}{3!\cdot 10} x^{10} + \cdots \] and therefore \[ T_8(x) = x + \frac{1}{1!\cdot 4} x^4 + \frac{1}{2!\cdot 7} x^7 \] Then \[ f(0.5) \approx T_8(0.5) = \frac{1}{2} + \frac{1}{4\cdot 2^4} + \frac{1}{14\cdot 2^7} = \frac{925}{1792} = 0.516183\ldots \]
Find a power series representation, centered at \(x=0\), for \[ f(x) = \frac{1}{3} (2x + x\sin(\tfrac{x}{5})) \] What is \(f^{(22)}(0)\)?\\
We have \begin{align*} f(x) &= \frac{1}{3}\left(2x + x\sumz \frac{(-1)^n (\tfrac{x}{5})^{2n+1}}{(2n+1)!}\right) \\[2ex] &= \frac{2}{3}x + \frac{x}{3} \sumz \frac{(-1)^n x^{2n+1}}{5^{2n+1}(2n+1)!} \\[2ex] &= \frac{2}{3}x + \sumz \frac{(-1)^n x^{2n+2}}{3\cdot 5^{2n+1}(2n+1)!} \end{align*} Then \[ f^{(22)}(0) = \frac{(-1)^{10}}{3 \cdot 5^{21} (21)!} \]
Find the Taylor series of \(f(x) = x e^x\) at \(a=1\). Then find \(T_3(x)\).
After successive differentiation one finds that \[ f^{(n)}(x) = ne^x + xe^x \] and thus \(f^{(n)}(1) = (n+1)e\). Thus the Taylor series is \[ \sumz \frac{(n+1)e}{n!} (x-1)^n \] and then \[ T_3(x) = e + 2e(x-1) + \frac{3e}{2!}(x-1)^2 + \frac{4e}{4!}(x-1)^3 \]
Find the Maclaurin series of \[ f(x) = \int_0^x \sin(t^2)\,dt \]
Not every function is equal to its Taylor series, that is, it isn't always true that \[ f(x) = \sumz \frac{f^{(n)}(a)}{n!}(x-a)^n \] For example, the function \[ f(x) = \begin{cases} e^{-1/x^2}, & x=0 \\[2ex] 0, & x=0 \end{cases} \] has derivatives at \(x=0\) all equal to zero, \(f^{(n)}(0) = 0\). Then the Taylor series is \[ \sumz \frac{f^{(n)}(0)}{n!} x^n = 0. \] This power series then converges on \((-\infty,\infty)\). However, it is clear that \(f(x)\) is not identically equal to zero:
figures/bump-function.svg